Skip to main content

Section 8.2 正项级数及其审敛性

研究级数的核心问题之一是如何判断级数的敛散性, 只有级数收敛, 才能对其进行求和、求近似值等运算. 而这里讨论的正项级数敛散性的判别方法是其他级数敛散性判别方法的基础.

Subsection 8.2.1 正项级数的基本性质

如果级数 \(\sum\limits_{k=1}^{\infty} u_{k}\) 中的每一项都是非负实数, 即 \(u_{n} \geqslant 0(n=1,2, \cdots)\text{,}\) 那么称该级数为正项级数. 正项级数是一类最基本的级数,同时也是一类重要的级数, 许多级数的敛散性问题往往可以归结为正项级数的敛散性问题.
\(\sum\limits_{k=1}^{\infty} u_{k}\) 为正项级数, \(s_{n}\) 是它的前 \(n\) 项部分和, 则有 \(s_{n+1}-s_{n}=u_{n+1} \geqslant 0(n=1\text{,}\) \(2, \cdots)\text{,}\) 因此正项级数的部分和数列 \(\left\{s_{n}\right\}\) 是单调递增的. 若数列 \(\left\{s_{n}\right\}\) 有上界, 则根据数列极限的单调有界准则, 数列 \(\left\{s_{n}\right\}\) 收敛, 从而正项级数收敛; 反之, 若正项级数收敛,则部分和数列 \(\left\{s_{n}\right\}\) 也收敛,由收敛数列必有界的性质可知, 数列 \(\left\{s_{n}\right\}\) 有上界. 因此可得下面正项级数收敛的基本定理.
下面考察另一个重要的级数— \(p\)-级数. \(p\)-级数和前面学过的调和级数、几何级数一样, 是作为判别其他级数收敛和发散的参照对象, 其结论的重要性是不言而喻的.
所谓 \(p\)-级数是指含参数 \(p\) 的如下级数
\begin{equation} \sum\limits_{n=1}^{\infty} \frac{1}{n^{p}}=1+\frac{1}{2^{p}}+\frac{1}{3^{p}}+\cdots+\frac{1}{n^{p}}+\cdots\tag{8.2.1} \end{equation}
其中 \(p>0, p\)-级数是正项级数. 前面已经证明: 当 \(p=1\) 时级数 (调和级数) 发散;当 \(p=2\) 时级数收敛. 对于一般情况, \(p\)-级数的敛散性有如下结论.

Example 8.2.2.

例 1 在 \(p\)-级数 (1) 中, 当 \(p \leqslant 1\) 时发散, 当 \(p>1\) 时收敛.
Solution.
证 当 \(p \leqslant 1\) 时, \(p\)-级数的部分和满足不等式
\begin{equation*} s_{n}=1+\frac{1}{2^{p}}+\frac{1}{3^{p}}+\cdots+\frac{1}{n^{p}} \geqslant 1+\frac{1}{2}+\frac{1}{3}+\cdots+\frac{1}{n}=\sigma_{n}, \end{equation*}
由 8.1 中Example 8.1.3 (调和级数发散) 的证明知 \(s_{n} \geqslant \sigma_{n}>\ln (1+n)\text{,}\) 所以部分和数列 \(\left\{s_{n}\right\}\) 无上界, 此时级数 \(\sum\limits_{n=1}^{\infty} \frac{1}{n^{p}}\) 发散.
下面证明: 当 \(p>1\) 时, \(p\)-级数收敛.
先将一般项 \(\frac{1}{n^{p}}\) 中的 \(n\) 换成连续自变量 \(x\text{,}\) 成为函数 \(\frac{1}{x^{p}}\text{.}\)\(1 \leqslant x<+\infty\) 上作函数\(y=\frac{1}{x^{p}}\)的图形,如图 8-1所示.于是\(p\)-级数从第二项到第\(n\)项的和\(\frac{1}{2^{p}}+\frac{1}{3^{p}}+\cdots+\frac{1}{n^{p}}\)就是图中有阴影的台阶形的面积,该台阶形的面积小于图中 \([1, n]\) 上的曲边梯形面积, 于是 \(p\)-级数的部分和满足
\begin{equation*} \begin{aligned} s_{n} & =1+\left(\frac{1}{2^{p}}+\frac{1}{3^{p}}+\cdots+\frac{1}{n^{p}}\right)<1+\displaystyle \int_{1}^{n} \frac{1}{x^{p}} \mathrm{~d} x \\ & =1+\left.\frac{1}{1-p} x^{1-p}\right|_{1} ^{n} \\ & =1+\frac{1}{p-1}\left(1-\frac{1}{n^{p-1}}\right)<1+\frac{1}{p-1}=M . \end{aligned} \end{equation*}
\(s_{n}\) 有上界, 所以当 \(p>1\)\(p\)-级数收敛.
归纳上述讨论的结果得
\begin{equation*} \sum\limits_{n=1}^{\infty} \frac{1}{n^{p}} \text { 的敛散性为 } \begin{cases}\text { 收敛, } & p>1, \\ \text { 发散, } & p \leqslant 1 .\end{cases} \end{equation*}
Example 8.2.2可知, 级数 \(\sum\limits_{n=1} \frac{1}{n^{\frac{3}{2}}}\) 收敛,而级数 \(\sum\limits_{n=1}^{\infty} \frac{1}{n^{\frac{1}{2}}}\) 发散.
Theorem 8.2.1很少直接用于正项级数敛散性的判定, 但它具有较高的理论价值, 正项级数的许多审敛法都是在它的基础上导出的. 下面介绍几种常用的正项级数的审敛法.

Subsection 8.2.2 正项级数的比较审敛法

Proof.

证 (1) 由 \(\sum\limits_{n=1}^{\infty} v_{n}\) 收敛知, 部分和 \(v_{1}+v_{2}+v_{3}+\cdots+v_{n}\) 有上界 \(M\text{.}\) 因为 \(0 \leqslant s_{n}=u_{1}+u_{2}+u_{3}+\cdots+u_{n} \leqslant v_{1}+v_{2}+v_{3}+\cdots+v_{n} \leqslant M(n=1,2,3, \cdots)\text{,}\)所以 \(s_{n}\) 也有上界, 从而正项级数 \(\sum\limits_{n=1}^{\infty} u_{n}\) 也收敛.
(2)由 \(\sum\limits_{n=1}^{\infty} v_{n}\) 发散知,部分和 \(v_{1}+v_{2}+v_{3}+\cdots+v_{n}\) 无上界. 因为 \(s_{n}=u_{1}+u_{2}+u_{3}+\cdots+u_{n} \geqslant v_{1}+v_{2}+v_{3}+\cdots+v_{n}(n=1,2,3, \cdots)\text{,}\)
所以 \(s_{n}\) 也无上界, 从而正项级数 \(\sum\limits_{n=1}^{\infty} u_{n}\) 发散.
使用比较审敛法Theorem 6.6.2时, 前面介绍的 3 个重要级数 (几何级数、 \(p\)-级数、调和级数)常常作为比较对象.

Example 8.2.4.

例 2 已知 \(a>1\text{,}\) 判别级数 \(\sum\limits_{n=1}^{\infty} \frac{1}{a^{n}+1}\) 的敛散性.
Solution.
解 由于 \(u_{n}=\frac{1}{a^{n}+1}<\left(\frac{1}{a}\right)^{n}=v_{n}(n=1,2,3, \cdots)\text{,}\)\(a>1\text{,}\) 所以 \(\frac{1}{a}<1, \sum\limits_{n=1}^{\infty} v_{n}\)是公比 \(q<1\) 的几何级数, 从而级数 \(\sum\limits_{n=1}^{\infty} v_{n}\) 收敛.
由比较审敛法知, \(\sum\limits_{n=1}^{\infty} \frac{1}{a^{n}+1}\) 收敛.

Example 8.2.5.

例 3 判别正项级数 \(\sum\limits_{n=1}^{\infty} \frac{1}{\sqrt{n(n+2)}}\) 的敛散性.
Solution.
解 由 \(n(n+2) \leqslant(n+1)^{2}\text{,}\)\(u_{n}=\frac{1}{\sqrt{n(n+2)}} \geqslant \frac{1}{n+1}(n=1,2,3, \cdots)\text{,}\) 而级数
\begin{equation*} \sum\limits_{n=1}^{\infty} \frac{1}{n+1}=\frac{1}{2}+\frac{1}{3}+\cdots+\frac{1}{n+1}+\cdots \end{equation*}
发散,所以 \(\sum\limits_{n=1}^{\infty} \frac{1}{\sqrt{n(n+2)}}\) 发散.
注意到级数各项同乘以一个非零常数 \(k\) 不会影响级数的敛散性,级数去掉或添加有限项也不会影响级数的敛散性,因此,由比较审敛法可得以下推论.

Example 8.2.7.

例 4 判别正项级数 \(\sum\limits_{n=1}^{\infty} \frac{\ln n}{n}\) 的敛散性.
Solution.
解 显然从第三项后, 即 \(n \geqslant 3\) 时有 \(\frac{\ln n}{n}>\frac{1}{n}\text{,}\)\(\sum\limits_{n=1}^{\infty} \frac{1}{n}\) 发散, 所以 \(\sum\limits_{n=1}^{\infty} \frac{\ln n}{n}\) 发散.
由于比较审敛法需要注意某一项后每项的不等式关系,在 \(u_{n}\) 的表示式复杂的情况下使用起来很不方便. 这里将以比较审敛法为基础给出一个更实用的审敛法。

Proof.

证 (1) 由极限的定义可知,对于 \(\varepsilon=1\text{,}\) 存在自然数 \(N\text{,}\)\(n>N\) 时, 有不等式 \(\left|\frac{u_{n}}{v_{n}}-l\right|<1\text{,}\) 因此当 \(n>N\) 时就有 \(u_{n}<(l+1) v_{n}\text{,}\) 根据Corollary 8.2.6即得所要证的结论.
(2)由已知条件知极限 \(\lim\limits_{n \rightarrow \infty} \frac{v_{n}}{u_{n}}=l(0 \leqslant l<+\infty)\text{,}\) 若级数 \(\sum\limits_{n=1}^{\infty} u_{n}\) 收敛, 则由Corollary 8.2.8(1) 知必有级数 \(\sum\limits_{n=1}^{\infty} v_{n}\) 收敛, 这与级数 \(\sum\limits_{n=1}^{\infty} v_{n}\) 发散的条件矛盾, 因此级数 \(\sum\limits_{n=1}^{\infty} u_{n}\) 不可能收敛, 即级数 \(\sum\limits_{n=1}^{\infty} u_{n}\) 发散.
Corollary 8.2.8说明, 如果 \(\lim\limits_{n \rightarrow \infty} \frac{u_{n}}{v_{n}}=l(0<l<+\infty)\text{,}\) 特别地当 \(n \rightarrow \infty, u_{n}\)\(v_{n}\) 是等价无穷小量 ( 即 \(\lim\limits_{n \rightarrow \infty} \frac{u_{n}}{\mathcal{V}_{n}}=1\) ) 时, 那么这两个正项级数 \(\sum\limits_{n=1}^{\infty} u_{n}\)\(\sum\limits_{n=1}^{\infty} v_{n}\) 同收敛或同发散.

Example 8.2.9.

例 5 判别 \(\sum\limits_{n=1}^{\infty} \sin \frac{\pi}{2^{n}}\)\(\sum\limits_{n=1}^{\infty} \ln \left(\frac{n+2}{n+1}\right)\) 的敛散性.
Solution.
解 当 \(n \rightarrow \infty\) 时, \(\sin \frac{\pi}{2^{n}} \sim \frac{\pi}{2^{n}}, \ln \left(1+\frac{1}{n+1}\right) \sim \frac{1}{n+1}\text{,}\)\(\sum\limits_{n=1}^{\infty} \frac{\pi}{2^{n}}\) 收敛, \(\sum\limits_{n=1}^{\infty} \frac{1}{n+1}\)发散,所以 \(\sum\limits_{n=1}^{\infty} \sin \frac{\pi}{2^{n}}\) 收敛, \(\sum\limits_{n=1}^{\infty} \ln \left(\frac{n+2}{n+1}\right)\) 发散.
Corollary 8.2.8 中令 \(v_{n}=\frac{1}{n^{p}}(n=1,2, \cdots)\text{,}\)\(\sum\limits_{n=1}^{\infty} u_{n}\)\(p\)-级数作比较, 根据 \(p\)-级数 的敛散性, 可以得到以下结论.

Example 8.2.11.

例 6 判别 \(\sum\limits_{n=1}^{\infty} \frac{n+1}{\sqrt{n^{3}+2 n+1}}\) 的敛散性.
Solution.
解 取 \(p=\frac{1}{2}\text{,}\) 由于 \(\lim\limits_{n \rightarrow \infty} n^{\frac{1}{2}} \cdot \frac{n+1}{\sqrt{n^{3}+2 n+1}}=1>0\text{,}\) 所以级数 \(\sum\limits_{n=1}^{\infty} \frac{n+1}{\sqrt{n^{3}+2 n+1}}\)发散.

Example 8.2.12.

例 7 判别 \(\sum\limits_{n=1}^{\infty} \sin \frac{\pi}{n^{2}}\) 的敛散性.
Solution.
解 取 \(p=2>1\text{,}\) 由于 \(\lim\limits_{n \rightarrow \infty} n^{2} \sin \frac{\pi}{n^{2}}=\pi>0\text{,}\) 所以级数 \(\sum\limits_{n=1}^{\infty} \sin \frac{\pi}{n^{2}}\) 收敛.

Subsection 8.2.3 正项级数的比值审敛法

用比较审敛法判别级数 \(\sum\limits_{n=1}^{\infty} u_{n}\) 的敛散性时都需要依赖于另一个级数 \(\sum\limits_{n=1}^{\infty} v_{n}\) 的敛散性, 这往往不太方便, 为此达朗贝尔利用比较审敛法给出了下面的比值审敛法. 它用级数自身的特性来判定其敛散性.

Proof.

证 (1) 因为 \(\lim\limits_{n \rightarrow \infty} \frac{u_{n+1}}{u_{n}}=\rho<1\text{,}\) 根据极限的定义, 对于 \(\varepsilon=\frac{1-\rho}{2}>0\text{,}\) 存在自然数 \(N\text{,}\)\(n>N\) 时有 \(\left|\frac{u_{n+1}}{u_{n}}-\rho\right|<\frac{1-\rho}{2}\text{,}\) 从而当 \(n>N\) 时有 \(0 \leqslant \frac{u_{n+1}}{u_{n}}<\frac{1+\rho}{2}=r<1\text{,}\) 因此
\begin{equation*} u_{N+2}<r u_{N+1}, u_{N+3}<r u_{N+2}<r^{2} u_{N+1}, u_{N+4}<r u_{N+3}<r^{3} u_{N+1}, \cdots . \end{equation*}
因为正项几何级数
\begin{equation*} u_{N+1}+u_{N+1} r+u_{N+1} r^{2}+u_{N+1} r^{3}+\cdots \quad(r<1) \end{equation*}
是收敛的, 所以由比较审敛法, 级数
\begin{equation*} u_{N+1}+u_{N+2}+u_{N+3}+u_{N+4}+\cdots \end{equation*}
也收敛. 在它的前面再添加有限项 ( \(N\) 项)
\begin{equation*} u_{1}+u_{2}+\cdots+u_{N} \end{equation*}
后的级数 \(\sum\limits_{n=1}^{\infty} u_{n}\) 不会改变敛散性,故 \(\sum\limits_{n=1}^{\infty} u_{n}\) 收敛.
(2)设 \(\rho>1\text{,}\) 因为 \(n \rightarrow \infty\) 时, \(\frac{u_{n+1}}{u_{n}} \rightarrow \rho>1\text{,}\) 存在自然数 \(N\text{,}\)\(n>N\) 时有 \(\left|\frac{u_{n+1}}{u_{n}}-\rho\right|<\frac{\rho-1}{2}\text{,}\) 所以当 \(n>N\) 时有 \(\frac{u_{n+1}}{u_{n}}>\frac{\rho+1}{2}=r>1\text{,}\) 从而可以推得, 当 \(n>N\)时有 \(u_{n+1}>u_{n} r>u_{n-1} r^{2}>\cdots>u_{N+1} r^{n-N}\text{,}\)\(r>1\)\(\lim\limits_{n \rightarrow \infty} u_{n}=+\infty\text{.}\) 因此, 由级数收敛的必要条件可知级数 \(\sum\limits_{n=1}^{\infty} u_{n}\) 发散.
(3)当 \(\rho=1\text{,}\)\(\lim\limits_{n \rightarrow \infty} \frac{u_{n+1}}{u_{n}}=1\) 时, 级数可能收敛也可能发散. 例如 \(p\)-级数, 不论 \(p\) 为何值都有
\begin{equation*} \lim\limits_{n \rightarrow \infty} \frac{u_{n+1}}{u_{n}}=\lim\limits_{n \rightarrow \infty} \frac{\frac{1}{(1+n)^{p}}}{\frac{1}{n^{p}}}=1 \end{equation*}
但是, 当 \(p>1\) 时级数收敛, 当 \(p \leqslant 1\) 时级数发散, 因此当 \(\rho=1\) 不能判别级数的敛散性.

Example 8.2.14.

例 8 判别下列级数的敛散性:
(1)\(\sum\limits_{n=1}^{\infty} \frac{a^{n}}{n !}(a>0)\text{;}\) (2) \(\sum\limits_{n=1}^{\infty} \frac{n !}{5^{n}}\text{;}\) (3) \(\sum\limits_{n=1}^{\infty} \frac{n !}{n^{n}}\text{.}\)
Solution.
解 (1) 由于
\begin{equation*} \lim\limits_{n \rightarrow \infty} \frac{u_{n+1}}{u_{n}}=\lim\limits_{n \rightarrow \infty}\left[\frac{a^{n+1}}{(n+1) !} \times \frac{n !}{a^{n}}\right]=\lim\limits_{n \rightarrow \infty} \frac{a}{n+1}=0<1, \end{equation*}
所以该级数收敛.
(2)由于
\begin{equation*} \lim\limits_{n \rightarrow \infty} \frac{u_{n+1}}{u_{n}}=\lim\limits_{n \rightarrow \infty}\left[\frac{(n+1) !}{5^{n+1}} \times \frac{5^{n}}{n !}\right]=\lim\limits_{n \rightarrow \infty} \frac{n+1}{5}=+\infty, \end{equation*}
所以该级数发散.
(3) 由于
\begin{equation*} \lim\limits_{n \rightarrow \infty} \frac{u_{n+1}}{u_{n}}=\lim\limits_{n \rightarrow \infty}\left[\frac{(n+1) !}{(n+1)^{n+1}} \times \frac{n^{n}}{n !}\right]=\lim\limits_{n \rightarrow \infty}\left(\frac{n}{n+1}\right)^{n}=\lim\limits_{n \rightarrow \infty} \frac{1}{\left(1+\frac{1}{n}\right)^{n}}=\frac{1}{\mathrm{e}}<1, \end{equation*}
所以该级数收敛.
另外, 根据级数收敛的必要条件 (8.1 Theorem 8.1.5) 还可以得到 \(\lim\limits_{n \rightarrow \infty} \frac{a^{n}}{n !}=0\) (其中 \(a>\) 0), \(\lim\limits_{n \rightarrow \infty} \frac{n !}{n^{n}}=0\text{.}\)

Example 8.2.15.

例 9 判别正项级数 \(\sum\limits_{n=1}^{\infty} \frac{(\lambda-\mathrm{e})^{2} \lambda^{n} \cdot n !}{n^{n}}\) 的敛散性 \((\lambda>0)\text{.}\)
Solution.
解 设 \(u_{n}=\frac{(\lambda-\mathrm{e})^{2} \lambda^{n} \cdot n !}{n^{n}}\text{,}\)\(\lambda \neq \mathrm{e}\) 时有 \(u_{n}>0(n=1,2,3, \cdots)\text{,}\)
\(\frac{u_{n+1}}{u_{n}}=u_{n+1} \cdot \frac{1}{u_{n}}=\frac{(\lambda-\mathrm{e})^{2} \lambda^{n+1} \cdot(n+1) !}{(n+1)^{n+1}} \cdot \frac{n^{n}}{(\lambda-\mathrm{e})^{2} \lambda^{n} \cdot n !}=\lambda\left(\frac{n}{n+1}\right)^{n}=\frac{\lambda}{\left(1+\frac{1}{n}\right)^{n}}\text{,}\)
所以
\begin{equation*} \lim\limits_{n \rightarrow \infty} \frac{u_{n+1}}{u_{n}}=\lim\limits_{n \rightarrow \infty} \frac{\lambda}{\left(1+\frac{1}{n}\right)^{n}}=\frac{\lambda}{\mathrm{e}} . \end{equation*}
因此, (1) 当 \(0<\lambda<\mathrm{e}\) 时,级数收敛;
(2)当 \(\lambda=\mathrm{e}\) 时,有 \(u_{n}=0(n=1,2, \cdots)\text{,}\) 级数收敛;
(3)当 \(\lambda>\mathrm{e}\) 时,级数发散.

Example 8.2.16.

例 10 证明级数 \(1+\frac{1}{1}+\frac{1}{1 \cdot 2}+\frac{1}{1 \cdot 2 \cdot 3}+\cdots+\frac{1}{1 \cdot 2 \cdot 3 \cdot \cdots \cdot(n-1)}+\cdots\) 是收敛的,并估计以级数的部分和 \(s_{n}\) 近似代替和 \(s\) 所产生的误差 \(\left|r_{n}\right|\text{.}\)
Solution.
解 因为
\begin{equation*} \begin{aligned} & \frac{u_{n+1}}{u_{n}}=\frac{1}{1 \cdot 2 \cdot 3 \cdot \cdots \cdot n} / \frac{1}{1 \cdot 2 \cdot 3 \cdot \cdots \cdot(n-1)}=\frac{1}{n}, \\ & \lim\limits_{n \rightarrow \infty} \frac{u_{n+1}}{u_{n}}=\lim\limits_{n \rightarrow \infty} \frac{1}{n}=0<1, \end{aligned} \end{equation*}
根据比值审敛法Theorem 8.2.13可知所给级数收敛.
以级数的部分和 \(s_{n}\) 近似代替和 \(s\) 所产生的误差为
\begin{equation*} \begin{aligned} \left|r_{n}\right| & =\frac{1}{n !}+\frac{1}{(n+1) !}+\frac{1}{(n+2) !}+\cdots \\ & =\frac{1}{n !}\left[1+\frac{1}{n+1}+\frac{1}{(n+1)(n+2)}\right]+\cdots \\ & <\frac{1}{n !}\left(1+\frac{1}{n}+\frac{1}{n^{2}}+\cdots\right)=\frac{1}{n !} \cdot \frac{1}{1-\frac{1}{n}}=\frac{1}{(n-1) \cdot(n-1) !} . \end{aligned} \end{equation*}
由此可见, 当 \(n\) 足够大时, 误差 \(\left|r_{n}\right|\) 可小于给出的任何小正数.

Example 8.2.17.

例 11 判别级数 \(\sum\limits_{n=1}^{\infty} \frac{1}{(2 n-1) \cdot 2 n}\) 的敛散性.
Solution.
\begin{equation*} \lim\limits_{n \rightarrow \infty} \frac{u_{n+1}}{u_{n}}=\lim\limits_{n \rightarrow \infty} \frac{(2 n-1) \cdot 2 n}{(2 n+1)(2 n+2)}=1, \end{equation*}
这时 \(\rho=1\text{,}\) 比值法失效(当 \(u_{n}\)\(n\) 的有理式时常出现这种情况), 改用其他方法.
因为 \(2 n>2 n-1 \geqslant n\text{,}\) 所以 \(u_{n}=\frac{1}{(2 n-1) \cdot 2 n}<\frac{1}{n^{2}}=v_{n}\text{,}\)\(\sum\limits_{n=1}^{\infty} v_{n}\)\(p=2\)\(p\)-级数, 收敛. 由比较法知原级数收敛.
最后举一个例子说明比值法常与比较法结合使用.

Example 8.2.18.

例 12 判别级数 \(\sum\limits_{n=1}^{\infty} \frac{n^{3}\left[\sqrt{2}+(-1)^{n}\right]^{n}}{3^{n}}\) 的敛散性.
Solution.
解 注意此级数为正项级数. 考虑到一般项 \(u_{n}\) 中含有 \((-1)^{n}\text{,}\) 先对此级数使用比较判别法 \(\frac{n^{3}\left[\sqrt{2}+(-1)^{n}\right]^{n}}{3^{n}} \leqslant \frac{n^{3}[\sqrt{2}+1]^{n}}{3^{n}}=v_{n}\text{,}\) 然后对级数 \(\sum\limits_{n=1}^{\infty} v_{n}\) 使用比值审敛法, 此时
\begin{equation*} \begin{aligned} \lim\limits_{n \rightarrow \infty} \frac{v_{n+1}}{v_{n}} & =\frac{(n+1)^{3}[\sqrt{2}+1]^{n+1}}{3^{n+1}} \cdot \frac{3^{n}}{n^{3}[\sqrt{2}+1]^{n}} \\ & =\lim\limits_{n \rightarrow \infty}\left(\frac{n+1}{n}\right)^{3} \cdot \frac{\sqrt{2}+1}{3}=\frac{\sqrt{2}+1}{3}<1 . \end{aligned} \end{equation*}
所以级数 \(\sum\limits_{n=11}^{\infty} \frac{n^{3}[\sqrt{2}+1]^{n}}{3^{n}}\) 收敛,由比较法知原级数收敛.

Subsection 8.2.4 正项级数的根值审敛法

利用正项级数的比较审敛法还可以推出下面正项级数的根值审敛法.

Proof.

证 (1) 因为 \(\lim\limits_{n \rightarrow \infty} \sqrt[n]{u_{n}}=\rho<1\text{,}\) 由数列极限定义可得, 对于 \(\varepsilon=\frac{1-\rho}{2}>0\text{,}\) 存在
\(N>0\text{,}\)\(n>N\) 时, \(\left|\sqrt[n]{u_{n}}-\rho\right|<\frac{1-\rho}{2}\text{,}\) 所以当 \(n>N\) 时有
\begin{equation*} 0 \leqslant \sqrt[n]{u_{n}}<\frac{1-\rho}{2}+\rho=\frac{1+\rho}{2}=r<1, \end{equation*}
即当 \(n>N\) 时有
\begin{equation*} 0 \leqslant u_{n}<r^{n} . \end{equation*}
\(0 \leqslant r<1\) 时, \(r^{N+1}+r^{N+2}+r^{N+3}+\cdots\) 收敛, 由比较审敛法得级数 \(\sum\limits_{k=1}^{\infty} u_{N+k}\) 收敛, 从而 \(\sum\limits_{n=1}^{\infty} u_{n}\) 收敛.
(2)当 \(\rho>1\) 时, 由数列极限定义可得, 对于 \(\varepsilon=\frac{\rho-1}{2}>0\text{,}\) 存在 \(N>0\text{,}\)\(n>N\)时, \(\left|\sqrt[n]{u_{n}}-\rho\right|<\frac{\rho-1}{2}\text{,}\) 从而此时有
\begin{equation*} \sqrt[n]{u_{n}}>-\frac{\rho-1}{2}+\rho=\frac{\rho+1}{2}=r>1, \end{equation*}
\begin{equation*} u_{n}>r^{n}(r>1), \end{equation*}
所以 \(\lim\limits_{n \rightarrow \infty} u_{n}=+\infty \neq 0, \sum\limits_{n=1}^{\infty} u_{n}\) 发散.
(3)当 \(\rho=1\) 时, 如 \(p\)-级数 \(\sum\limits_{n=1}^{\infty} \frac{1}{n^{p}}\text{,}\)\(\sqrt[n]{u_{n}} \rightarrow 1(n \rightarrow \infty)\text{.}\) 但当 \(p=1\) 时, 即 \(\sum\limits_{n=1}^{\infty} \frac{1}{n}\)是发散级数, \(p=2\) 时, 即 \(\sum\limits_{n=1}^{\infty} \frac{1}{n^{2}}\) 是收敛级数. 这说明当 \(\rho=1\) 时级数可能收敛也可能发散.

Example 8.2.20.

例 13 判别下列级数的敛散性: (1) \(\sum\limits_{n=1}^{\infty}\left(\frac{n}{2 n+1}\right)^{n}\text{;}\) (2) \(\sum\limits_{n=1}^{\infty} \frac{3+(-1)^{n}}{3^{n}}\text{.}\)
Solution.
解 (1) 这里 \(u_{n}=\left(\frac{n}{2 n+1}\right)^{n}\text{,}\) 由于 \(\sqrt[n]{u_{n}}=\sqrt[n]{\left(\frac{n}{2 n+1}\right)^{n}}=\frac{n}{2 n+1} \rightarrow \frac{1}{2}<1(n \rightarrow \infty)\text{,}\)所以该级数收敛.
(2)由于 \(\lim\limits_{n \rightarrow \infty} \sqrt[n]{u_{n}}=\lim\limits_{n \rightarrow \infty} \frac{1}{3} \sqrt[n]{3+(-1)^{n}}=\frac{1}{3}<1\text{,}\) 由根值审敛法知, 该级数收敛.

Example 8.2.21.

例 14 设 \(a_{n}, b, a\) 均为正数, 且 \(a_{n} \rightarrow a(n \rightarrow \infty)\text{,}\) 试判别级数 \(\sum\limits_{n=1}^{\infty}\left(\frac{b}{a_{n}}\right)^{n}\) 的敛散性.
Solution.
解 因为 \(\lim\limits_{n \rightarrow \infty} \sqrt[n]{u_{n}}=\lim\limits_{n \rightarrow \infty} \sqrt[n]{\left(\frac{b}{a_{n}}\right)^{n}}=\lim\limits_{n \rightarrow \infty} \frac{b}{a_{n}}=\frac{b}{a}\text{,}\) 需要讨论参数 \(a, b\) 的不同情况.
\(b<a\text{,}\)\(\frac{b}{a}<1\) 时, 级数收敛; 当 \(b>a\text{,}\)\(\frac{b}{a}>1\) 时, 级数发散;
\(b=a\text{,}\)\(\frac{b}{a}=1\) 时, 级数 \(\sum\limits_{n=1}^{\infty}\left(\frac{b}{a}\right)^{n}\) 可能收敛也可能发散. 例如, 若 \(a_{n}=\sqrt[n]{n^{2}}, b=\) 1, 则 \(a=\lim\limits_{n \rightarrow \infty} a_{n}=1, \sum\limits_{n=1}^{\infty}\left(\frac{b}{a_{n}}\right)^{n}=\sum\limits_{n=1}^{\infty} \frac{1}{n^{2}}\text{;}\)\(a_{n}=\sqrt[n]{n}, b=1\text{,}\)\(a=\lim\limits_{n \rightarrow \infty} a_{n}=1\text{,}\) \(\sum\limits_{n=1}^{\infty}\left(\frac{b}{a_{n}}\right)^{n}=\sum\limits_{n=1}^{\infty} \frac{1}{n}\) 发散.

Subsection 8.2.5 *正项级数的积分审敛法

数项级数 \(\sum\limits_{n=1}^{\infty} u_{n}\) 求和实际上可以理解为分段函数 \(f(x)=u_{n}(x \in[n, n+1))\) 在区间 \([1,+\infty)\) 上的无穷积分. 因此,级数 \(\sum\limits_{n=1}^{\infty} u_{n}\) 的基本性质和审敛法, 与 \([1,+\infty)\) 上无穷积分的基本性质和审敛法有着十分密切的联系. 下面给出一种根据无穷积分的敛散性判定正项级数敛散性的方法.

Proof.

证 首先注意,因为 \(f(x)\) 单调下降, \(f(n)=u_{n} \geqslant 0\text{,}\)所以 \(f(x) \geqslant 0\text{.}\) 考虑 \(\displaystyle \int_{1}^{+\infty} f(x) \mathrm{d} x\) 的几何意义, 它是由曲线 \(y=f(x), x=1\)\(x\) 轴围成的向右无限伸展的图形的面积 (见图 8-2).
(1)设 \(\displaystyle \int_{1}^{+\infty} f(x) \mathrm{d} x\) 收敛. 假设 \(\displaystyle \int_{1}^{+\infty} f(x) \mathrm{d} x=K\) \((K<+\infty)\text{,}\)\(x=1,2,3, \cdots\) 处作 \(x\) 轴的垂线到曲线
\(y=f(x)\) 为止,分别交曲线于 \(A_{1}, A_{2}, A_{3}, \cdots\text{,}\)再分别从 \(A_{2}, A_{3}, \cdots\text{,}\) 向左作 \(x\) 轴的平行线而构成许多矩形 (如图 8-2 所示). 注意它们的高分别为 \(f(2)=u_{2}, f(3)=\) \(u_{3}, \cdots, f(n)=u_{n}, \cdots\text{,}\) 而底宽都等于 1 , 且 \(f(x)\) 是单调下降的. 根据积分的几何意义, 可以看出
\begin{equation*} u_{2}+u_{3}+\cdots+u_{n} \leqslant \displaystyle \int_{1}^{n} f(x) \mathrm{d} x \leqslant \displaystyle \int_{1}^{+\infty} f(x) \mathrm{d} x=K(K<+\infty) . \end{equation*}
所以正项级数 \(\sum\limits_{n=1}^{\infty} u_{n}\) 的部分和数列有上界, 从而 \(\sum\limits_{n=1}^{\infty} u_{n}\) 收敛.
(2)用反证法. 假设级数 \(\sum\limits_{n=1}^{\infty} u_{n}\) 收敛, 类似于 (1)中作图, 但现在分别从 \(A_{1}, A_{2}, A_{3} \cdots\) 向右作 \(x\) 轴的平行线 (见图 8-3), 可以看出
\begin{equation*} \displaystyle \int_{1}^{n+1} f(x) \mathrm{d} x \leqslant s_{n}=u_{1}+u_{2}+u_{3}+\cdots+u_{n} \leqslant s \end{equation*}
其中 \(s\) 是级数 \(\sum\limits_{n=1} u_{n}\) 的和. 由无穷限反常积分的审敛法得 \(\displaystyle \int_{1}^{+\infty} f(x) \mathrm{d} x\) 发散, 而这与假设矛盾, 因此级数 \(\sum\limits_{n=1}^{\infty} u_{n}\) 发散.
下面举例说明积分审敛法的用法.

Example 8.2.23.

例 15 用积分审敛法讨论对数级数 \(\sum\limits_{n=2}^{\infty} \frac{1}{n(\ln n)^{k}}\) 的敛散性 \((k>0)\text{.}\)
Solution.
解 该级数可以看作 \(\sum\limits_{n=1}^{\infty} \frac{1}{(n+1)[\ln (n+1)]^{k}}\text{.}\)
\(f(x)=\frac{1}{(x+1)[\ln (x+1)]^{k}}\text{,}\) 显然 \(f(x)\)\([1,+\infty)\) 上单调递减, 并且 \(f(n)=\) \(u_{n}\text{.}\) 因为
(1) 当 \(k \neq 1\) 时,
\begin{equation*} \begin{aligned} \displaystyle \int_{1}^{+\infty} \frac{1}{(x+1)[\ln (x+1)]^{k}} \mathrm{~d} x & =\left.\lim\limits_{b \rightarrow+\infty}\left\{\frac{1}{1-k}[\ln (x+1)]^{1-k}\right\}\right|_{1} ^{b} \\ & = \begin{cases}\frac{(\ln 2)^{1-k}}{k-1}, & k>1, \\ +\infty, & k<1 .\end{cases} \end{aligned} \end{equation*}
(2)当 \(k=1\) 时, \(\displaystyle \int_{1}^{+\infty} \frac{\mathrm{d} x}{(x+1) \ln (x+1)}=\lim\limits_{b \rightarrow+\infty}[\ln |\ln (x+1)|]_{1}^{b}=+\infty\text{,}\) 发散.
于是, 对于对数级数 \(\sum\limits_{n=2}^{\infty} \frac{1}{n(\ln n)^{k}}\text{,}\)\(k>1\) 时,收敛; 当 \(k \leqslant 1\) 时,发散.

Subsection 8.2.6 习题 8-2

  1. 用比较审敛法判别下列正项级数的敛散性:
    1. \(\sum\limits_{n=1}^{\infty} \frac{1}{1+\sqrt{n}}\text{;}\)
    2. \(\sum\limits_{n=1}^{\infty} \frac{1+n}{1+n^{2}}\text{;}\)
    3. \(\sum\limits_{n=1}^{\infty} \frac{\ln n}{n}\text{;}\)
    4. \(\sum\limits_{n=1}^{\infty} 2^{n} \sin \frac{\pi}{3^{n}}\text{;}\)
    5. \(\sum\limits_{n=1}^{\infty} \frac{1}{2 n^{2}-n+1}\text{.}\)
  2. 用比值审敛法判别下列正项级数的敛散性:
    1. \(\sum\limits_{n=1}^{\infty} \frac{n !}{n^{n}}\text{;}\)
    2. \(\sum\limits_{n=1}^{\infty} n \tan \frac{\pi}{2^{n+1}}\text{;}\)
    3. \(\sum\limits_{n=1}^{\infty} \frac{(a+1)(2 a+1) \cdots(n a+1)}{(b+1)(2 b+1) \cdots(n b+1)}(a>0, b>0)\text{.}\)
  3. 用根值法判别下列正项级数敛散性:
    1. \(\sum\limits_{n=1}^{\infty}\left(1-\frac{1}{n}\right)^{n^{2}}\text{;}\)
    2. \(\sum\limits_{n=1}^{\infty} \frac{1}{2^{n}}\left(1+\frac{1}{n}\right)^{n^{2}}\text{;}\)
    3. \(\sum\limits_{n=1}^{\infty}\left(\frac{n}{3 n-1}\right)^{2 n-1}\text{;}\)
    4. \(\sum\limits_{n=1}^{\infty} \frac{n^{n-1}}{\left(2 n^{2}+n+1\right)^{\frac{n+1}{2}}}\text{.}\)
  4. 判别下列级数的敛散性:
    1. \(\sum\limits_{n=1}^{\infty}\left(1-\cos \frac{\pi}{n}\right)\text{;}\)
    2. \(\sum\limits_{n=1}^{\infty} \frac{1}{n \sqrt[n]{n}}\text{;}\)
    3. \(\sum\limits_{n=1}^{\infty} \frac{[(n+1) !]^{2}}{2 ! 4 ! \cdots(2 n) !}\text{;}\)
    4. \(\sum\limits_{n=1}^{\infty} \displaystyle \int_{0}^{\frac{1}{n}} \frac{\sqrt[3]{x}}{4+x^{2}} \mathrm{~d} x\text{;}\)
    5. \(\sum\limits_{n=1}^{\infty}(\sqrt{n+1}-\sqrt{n}) \sin \frac{4 \pi}{n}\text{;}\)
    6. \(\sum\limits_{n=1}^{\infty} \frac{a^{n}}{n^{s}}(a>0, s>0)\text{.}\)